glannon

Pataasin ang iyong marka sa homework at exams ngayon gamit ang Quizwiz!

QUESTION 4. Criticizing the jury. A judge presided over a controversial, high-profile jury trial of a county clerk who engaged in civil disobedience and refused to issue marriage licenses to gay couples, even after receiving a court order to do so. The judge thought that the law and the evidence against the defendant was overwhelming, as the clerk freely admitted that she had willfully flouted the law and disregarded the court order. Even so, the jury decided to engage in nullification and acquitted the county clerk of all charges. In shock, the judge chastised the jury for not honoring their oath to uphold the law, and for allowing their political views influence their verdict in the case. Under the Code of Judicial Conduct, which of the following is true?

A.The judge could be subject to discipline for criticizing the jurors for their decision.

QUESTION 10. Choose the violation. An attorney accepted a position on the state Commission on Judicial Conduct, a division of the judicial branch that investigates complaints of misconduct by judges and brings disciplinary actions when a violation of the Code of Judicial Conduct has occurred. The attorney was surprised by the wide range of rules that regulate the conduct of judges, as well as the variety of complaints that the Commission had to investigate each year. Which of the following activities violated the Code of Judicial Conduct?

B.A judge's clerk did Internet research about alternative methods for toxic mold remediation in homes in a case in which one of the parties alleged, among other things, that the opposing party failed to mitigate damage from toxic mold. The clerk wrote a memorandum for the judge about her research, and the judge did not disclose to the parties or their counsel that the clerk had done this research.

QUESTION 7. Fact-checking on the Internet. A traffic accident led to litigation. At trial, one of the witnesses explained that dense tree foliage blocked the visibility of drivers from seeing traffic that was turning into their lane from a side street. After court adjourned for the day, the judge checked the accident scene on Google Earth and discovered that the entire area had retail and commercial development along both sides of the road. Few, if any, trees remained in the area. Did the judge violate the Code of Judicial Conduct?

B.The judge's Internet search violated CJC Rule 2.9(c) because this is an adjudicative fact.

QUESTION 8. Background research. A newly appointed judge finds herself assigned to a court in a rural agricultural area, so the court has many cases related to farm ownership and foreclosures, government regulation and subsidies for agriculture, and so on. The judge has lived her entire life in a major urban center, and she is unfamiliar with this area of law and the economic realities faced by individuals and firms in the farming industry. To learn more background for upcoming cases that are not yet assigned, the new judge spends some of her weekends and evenings reading articles and blogs online. Does this background research violate the CJC?

C.No, because judges may do background reading to become better informed.

QUESTION 5. The mutual fund investments. A judge has investments in a Felicity Mutual Fund, a large mutual fund that invests in "blue chip" stocks of well-established, large corporations that offer lower risk than other stocks. A case arises on the judge's docket involving Intec, a large company whose stocks are currently among those in the Felicity Mutual Fund's portfolio. Lawyers for the opposing party file a motion for the judge to recuse or disqualify herself because of these investments. Which of the following describes what the Code of Judicial Conduct would require of the judge in this circumstance?

C.The judge does not need to disqualify herself, because the stock holdings in Intec are indirect investments made through a mutual fund.

QUESTION 9. Court records and archives. During the sentencing phase of a criminal trial, the judge grew concerned that the prosecutor expressed uncertainty about the number of the defendant's prior convictions. The judge instructed her clerk to search the court's electronic docket and archives to determine the defendant's criminal case history, and to expand the search to other jurisdictions if their records were accessible from the clerk's computer. The clerk discovered several other judicial proceedings involving the defendant, and carefully noted how many of these had resulted in convictions. The clerk also noted pending cases awaiting sentencing that might prompt the judge to wait a little longer before rendering a decision, so that the judge could consider those cases as well in her own sentencing. Finally, the clerk noted some cases with sealed records, which the clerk was able to access and review and report to the judge. The judge intended to follow appropriate procedures for taking judicial notice of any of these facts that might bear on the defendant's sentence. Was the clerk's search proper, according to the Code of Judicial Conduct?

D.It was appropriate for the clerk to review the court dockets and archives for the defendant's other proceedings but not to review the court records that were under seal.

QUESTION 6. The television commercial. A local furniture store decided to make a low-budget television commercial telling customers that the store was having its best clearance sale ever. The owner of the store lived next door to a state trial judge, who had held his judicial seat for twenty years and was well-known in the community. At the request of the furniture store owner, the judge agreed to appear in his judicial robe in the television commercial, recommending the store and its owner to the viewers. Could the judge be subject to discipline for this commercial, under the Code of Judicial Conduct?

D.Yes, because it violated the Code of Judicial Conduct for a judge to appear in judicial robes in business advertising.

QUESTION 11. Duties in the cloud. The attorney has switched to cloud computing, meaning that their firm pays a monthly fee to store all their spreadsheets and documents in an Internet-based database or archive. This protects client information and case documents from being lost whenever a computer at the firm crashes; the cloud service automatically creates an online backup for every file. According to the Rules of Professional Conduct, which of the following is true?

A.The attorney and his firm have an affirmative duty to make reasonable efforts to ensure that the cloud service is secure against computer hacking or other invasive access to clients' confidential information.

QUESTION 10. Regulation by the legislature. A state legislature enacted a statute governing the licensing of attorneys and discipline for practitioners. The preamble to the statute asserted "field preemption" over the regulation of lawyers in that jurisdiction, thereby abolishing all prior rules and codes of the state bar. An attorney came under discipline under the new law and contested the legal validity of the enactment itself. What should be the result?

A.The court will hold the law invalid because the judiciary has inherent power to regulate the attorneys who practice in its courts.

QUESTION 8. Reasonable resolution. An attorney is a new associate at a law firm, and the managing partner assigns her a new case, in which the firm will represent two co-plaintiffs in a personal injury case. The attorney is concerned that a conflict of interest could arise between the two plaintiffs and suggests that the firm should represent only one of them. When she discusses this with the managing partner, the managing partner disagrees, because the interests of the two plaintiffs seem perfectly aligned, and they can have each sign an informed consent form waiving the conflict up front. Both admit the question is a close one in terms of the ethical rules for conflicts of interest, but the managing partner insists that they proceed. Which of the following is true regarding this situation?

A.The supervisor's reasonable resolution of the question should protect the subordinate professionally if the resolution subsequently faces a challenge.

QUESTION 12. Have it your way. An attorney represents a sophisticated business client in a litigation matter. The attorney wants to hire an outside non-lawyer investigator/paraprofessional to help find and develop evidence and witnesses for the case. The client agrees but wants the attorney to hire a particular outside company with whom the client has close business dealings and a long history. The attorney would normally have used a different firm that is more familiar to him. Which of the following is correct, according to the Model Rules and the accompanying comments?

A.Where the client directs the selection of a specific non-lawyer service provider outside the firm, the lawyer ordinarily should agree with the client concerning the allocation of responsibility for monitoring as between the client and the lawyer.

QUESTION 8. Who is behind this report? A client wants to sell a parcel of commercial real estate, and he hired an attorney to represent him in the matter. As part of the representation, the client asked his attorney to prepare a thoroughly researched opinion memorandum concerning the title of the property, for the information of a prospective purchaser and the purchaser's prospective lender. The attorney gave the title opinion to the client, who gave it to the prospective purchaser, who in turn submitted it to the prospective lender. The prospective lender received and reviewed the attorney's title opinion but was not aware that the lawyer who prepared the title opinion represented the seller of the property rather than the buyer. Could the attorney be subject to discipline for failing to disclose explicitly in the memorandum what party he represents and that he has a duty of loyalty and confidentiality to the seller?

B.Yes, because the title opinion should identify the person by whom the lawyer is retained, and should make this clear not only to the client under examination, but also to others to whom the results are to be made available.

QUESTION 25. Non-compete agreements for associates. An attorney owns his own firm in a small town and hires an associate as a junior lawyer to help with the growing caseload. The employment agreement stipulates that the associate cannot practice law in that small town after leaving the attorney's firm. Which of the following is true regarding this arrangement?

C.Both the attorney and the associate are subject to discipline for such an agreement.

QUESTION 4. My client is a suspicious character. A client hired an attorney to represent him in a simple real estate matter. When the attorney asked some standard questions about the financial arrangements for the sale and purchase of the property, the client was somewhat evasive on a few points, but provided the information necessary to complete the legal work for the transaction. The attorney also heard from a friend that the client frequently cavorted with prostitutes. The attorney finds the client suspicious and has many unanswered questions, but none surrounding the transaction that occasioned the representation. Does the attorney have an ethical duty to inquire into the affairs of a suspicious client?

D.No, because a lawyer ordinarily has no duty to initiate investigation of a client's affairs or to give advice that the client has indicated is unwanted.

QUESTION 8. It's the little things. An attorney faced criminal sanctions for having over two thousand unpaid traffic and parking tickets, and several instances of failure to appear for jury duty. Could the attorney be subject to professional discipline for these minor offenses?

B.Yes, because a pattern of repeated offenses, even ones of minor significance when considered separately, can indicate indifference to legal obligation.

QUESTION 10. Certified by whom? An attorney is properly certified as an immigration law specialist by a state bar organization that provides official certifications. In her advertisements, the attorney describes herself as a "Certified Specialist in Immigration Law" without identifying the certifying organization. The attorney also mentions that she speaks Spanish and Portuguese (besides English), and that her fees are very affordable. Could the attorney be subject to discipline for making such statements in her advertisements?

A.Yes, because she failed to identify the certifying organization.

QUESTION 3. Respondeat superior. An attorney is a fifth-year associate at a large firm and is responsible for supervising the work of a first-year associate. The attorney, however, now spends most of his time in Singapore, trying to open a satellite office for the firm there to service one of its major corporate clients. He has not inquired into the associate's compliance with the Rules of Professional Conduct in over eighteen months, as they mostly communicate by e-mail regarding pending cases and assignments. To the best of his knowledge, though, the attorney believes the associate is following the rules, and he knows that the associate has attended two Legal Ethics CLE courses in the last year. Unbeknownst to the attorney, the new associate has been overbilling hours and has been neglecting certain client matters. Which of the following is true regarding the attorney's situation?

A.The attorney is subject to discipline as a lawyer having direct supervisory authority over another lawyer who failed to make reasonable efforts to ensure that the other lawyer conforms to the Rules of Professional Conduct, even though there was no direction, ratification, or knowledge of the violation.

QUESTION 2. A mutually beneficial arrangement. An attorney made an informal agreement with a physician that they would refer clients to each other when the situation seemed appropriate. They did not pay each other any money for referrals, but the relationship was explicitly reciprocal—the attorney referred patients who needed medical examinations to the physician, and when the physician had patients needing legal representation, he referred them to the attorney. The relationship was not explicitly exclusive—each was free to refer clients to others—but it happened that neither had similar reciprocal relationships with anyone else. They always informed their clients when making such referrals that they had a reciprocal relationship. Is such an arrangement proper?

A.Yes, a lawyer may agree to refer clients to another lawyer or a non-lawyer professional, in return for the undertaking of that person to refer clients or customers to the lawyer, as long as clients are aware and the relationship is not exclusive.

QUESTION 5. European vacation. An attorney received a small settlement check from a client for the opposing party on Christmas Eve, late in the afternoon. All the support staff at the firm had already gone home for the day, due to the impending holiday, and the firm was to stay closed until January 2. The attorney was rushing to catch a flight to Europe, where she planned to spend the holidays with her family. On January 2, when the attorney returned and reopened the office, she promptly notified the opposing party that the check had arrived. Could the attorney be subject to discipline for her actions in this case?

A.Yes, because she did not notify the opposing party soon enough.

QUESTION 5. Let's fix this. An attorney is a fifth-year associate at a large firm, hoping to make partner in the next two or three years. She supervises the first-year associates at the firm. She learns that the most recently hired associate recently shredded some evidence that would have undermined a client's case, and then told the judge and opposing party that the missing documents had been in a briefcase that went missing when a burglar broke into the associate's car. The attorney knows this is not true and discusses it with the senior litigation partner, who finds the story amusing. Neither reports the associate's deception to the judge or opposing party. Which of the following statements is true regarding this situation?

A.Both the attorney and the litigation partner are subject to discipline for not taking action to correct the associate's false statements and misconduct.

QUESTION 5. Contacting the warehouse worker. While preparing for a trial over workplace discrimination, the plaintiff's attorney contacts an entry-level, night-shift worker in the company's off-site warehouse, who supposedly told the frequent, shocking sex jokes that led to the "hostile environment" claim by female workers that became the subject of the pending lawsuit. The attorney did this without the permission of the company's lawyer, even though he suspects the company's lawyer would have allowed it if he had asked. The warehouse worker has no supervisory authority, has never been to the corporate offices, has no authority to bind the company, and is now on suspension (unpaid leave) while the company does its own internal investigation of the allegations. Was the communication by the plaintiff's attorney proper?

A.No, the attorney may not communicate with the constituent of a represented organization (opposing party) whose act or omission in connection with the matter may be imputed to the organization for purposes of civil liability.

QUESTION 2. Not my brother's keeper. An attorney is a second-year associate at a law firm with no supervisory responsibilities. He learns that another second-year associate is working on a case in which the client is suing a company that the other associate used to represent at his previous firm, and the attorney suspects it is a substantially related matter. The firm has done nothing to screen the other associate from the matter. No one ever discusses it with the attorney, and the attorney does not know all the facts of the situation. Later, the client sues the firm for malpractice due to the conflict of interest and reports the matter to the state disciplinary authority. Which of the following is true regarding the attorney's involvement in the situation?

A.The attorney does not have disciplinary liability for the conduct of the other associate, because he is neither a partner nor in a supervisory position and did not participate in the violations directly.

QUESTION 9. Nuremberg defense. An attorney is an associate in a litigation firm representing plaintiffs. In her current case, her supervising partner instructs her to assert that the defendant had an affirmative statutory duty to protect the plaintiff's interests, even though the attorney can find no statute to support this assertion. The attorney has brought this to the attention of her supervising partner, who rebuked her for questioning his authority and insisted that she do as he said. He assures her that the defendants will settle before trial anyway, so the bogus claim merely gives some psychological leverage during settlement negotiations, and it cannot do any harm. Moreover, the partner says that the attorney may not last long at the firm if she cannot follow instructions, which could have been a threat of termination. At a preliminary hearing, however, the judge confronts the attorney about the unsupportable claim, and she concedes that no statutory duty exists. The judge is irate and considers reporting the attorney to the state bar disciplinary authority. Which of the following is correct regarding the attorney's situation?

A.The attorney is responsible for asserting a frivolous claim, even though her supervising partner insisted that she do it and threatened her with termination.

QUESTION 4. Protocols in place. An attorney is a partner in a medium-size firm. Another partner at the firm, the managing partner, is responsible for implementing policies and procedures to detect and resolve conflicts of interest, to account for client funds and property, to identify dates by which actions must occur in pending matters, and to ensure that inexperienced lawyers receive proper supervision. The managing partner, however, now spends most of his time in Singapore, trying to open a satellite office for the firm there to service one of its major corporate clients. The managing partner is rarely at the home office and has completely neglected the implementation of ethical policies in the firm, so that minimal safeguards or procedures are in place. One of the new associates has committed several serious violations of professional responsibilities in the last few months, including an egregious conflict-of-interest problem and several missed deadlines for filing responsive pleadings. The attorney knew nothing about the violations and was not directly supervising the associate, and tries not to meddle in any of the managing partner's responsibilities, including the implementation of ethical policies and procedures. Which of the following is correct?

A.The attorney is subject to discipline as a partner in the firm for failing to make reasonable efforts to ensure that the firm has in effect measures giving reasonable assurance that all lawyers in the firm conform to the Rules of Professional Conduct.

QUESTION 7. Just following orders. The attorney is an associate at a small firm, and her supervising partner instructs her to draft pleadings in a case for a client. The supervising partner knows that the statute of limitations has already run on the claim, and that the client had little or no factual evidence to support the claim in any case. The partner believes the opposing party will want to settle the claim quickly for a modest sum and will not bother to investigate issues such as the statute of limitations or the factual support for either side. The attorney follows the partner's instructions and drafts the pleadings, without checking the statute of limitations for this specific claim or conducting her own investigation into the facts of the case. Opposing counsel, however, is upset over the frivolous claim and reports the attorney to the state bar. Which of the following is correct regarding the attorney's situation?

A.The attorney should not be responsible for asserting a frivolous claim, and the fact that she was just following orders could support her defense that she was unaware that the claim was frivolous.

QUESTION 26. The lawyer-CPA. An attorney specializes in estate planning. Besides being a lawyer, she is a certified public accountant (CPA). One of her clients hires her to prepare a will and handle the planning for a complex estate, which will involve creating two charitable trusts and other maneuvers for avoiding hefty estate taxes. The estate planning in this case involves some transfers to create the trusts in the current calendar year, which will be reportable on the current year tax returns. The client asks the attorney to prepare her tax returns for the current year, given that the attorney is handling all the estate planning, and already has all the documentation about the finances and assets of client. The attorney agrees to prepare the returns as a CPA, and she creates a separate retainer agreement with the client for the preparation of the tax returns, one that complies with all IRS requirements for tax preparers and stipulates this retainer shall be for accounting work, not legal services. Five years later, the same client runs for Congress, and during a contentious campaign, a reporter asks the attorney how much the client paid in taxes in the year that the attorney prepared the tax returns. The attorney answers the question in detail. The client complains that this constitutes a breach of lawyer confidentiality, but the attorney defends her actions by explaining that the amount of taxes paid that year was information derived solely from her work as a CPA, under a separate retainer with due disclosures, and not as a lawyer. Who is correct here—the client, or the attorney?

A.The client is correct because the circumstances were such that the non-legal accounting services were not distinct from the legal services attorney was providing at the time.

QUESTION 10. Relevant factors. A partner gives an associate the typed notes from a previous client interview conducted by the partner, and the associate has the task of drafting a complaint for a personal injury lawsuit based on the allegations in the notes. The associate has no direct contact with the client, and does not really have any way to verify whether the notes represent everything discussed in the interview (the notes are not a transcript) or whether the allegations are factually accurate, truthful, or tell the complete story. The associate completes the task as assigned, drafting the pleadings based on the notes. The associate then submits the drafted complaint to the partner for review. Later, the pleadings turn out to be frivolous, based on complete falsehoods. Which of the following is true regarding the associate attorney's role in drafting the complaint?

A.The lack of opportunity for the associate to investigate or verify facts on her own will be a relevant factor for the state disciplinary authority in deciding whether to discipline the associate.

QUESTION 6. Dumping on the associates. An attorney works at the state Public Defender office. Due to their insufficient funding and the overwhelming number of indigent defendants in her city, her caseload is so great that she cannot do adequate investigation into any of her client's cases or conduct legal research about possible defenses. Nevertheless, 95 percent of the cases end in plea bargains without going to trial, so the attorney tells herself that her neglect of case development makes no difference. Her supervisor at the Public Defender office is aware of the unreasonable caseloads of all the attorneys who work there, but the supervisor wants the attorneys to increase their caseloads in order to provide representation to more indigent defendants, even if that means doing minimal work on each case. Which of the following is true regarding the ethical situation facing the attorney and her supervisor?

A.The supervisor could be subject to discipline for not ensuring that a subordinate attorney can manage her workload, even if that means not assigning the lawyer any more cases for now.

QUESTION 1. Responsibility versus liability. An attorney is a partner in a newer law firm that has no effective measures in place to ensure that lawyers in the firm conform to the Rules of Professional Conduct. An associate at the firm violates the rules, and the state bar investigates the policies and procedures in place at the firm. The state disciplinary authority has determined that the first attorney is subject to discipline for his failure to take reasonable measures to ensure conformity with the rules. Because of this determination and the subsequent sanction, which of the following is true?

A.Whether a lawyer may be liable civilly or criminally for another lawyer's conduct is a question of law beyond the scope of the rules; the determination of a violation does not automatically mean that the partner attorney would be civilly or criminally liable.

QUESTION 15. A deeply divided law school faculty. A law school suffers from deep divisions among its faculty. One group of the faculty dislikes the Dean and wants to force his resignation with a vote of no confidence and pressure on the Board of Trustees. The other group is loyal to the Dean and resents their disloyal colleagues, whom they consider unprofessional. The controversy surrounding the law school's Dean overlaps with faculty divisions over hiring practices, tenure, and whether the school should try to emulate top-tier law schools in order to boost their national rankings, or if they should focus instead exclusively on pedagogy and preparing the students for the practice of law after graduation. The divisions are so great that each faction has threatened to quit or take other drastic action that would imperil the school's existence, if their side does not prevail. The Board of Trustees obtains an agreement from both factions on the faculty that they will hire an attorney to function as a third-party neutral to attempt to broker a compromise between the factions on the faculty. The attorney is an alumnus of the law school and offers to serve in this capacity without charging legal fees. He claims that he is not representing the Board, the Dean, or either side of the balkanized faculty. He begins to schedule private conferences with each faculty group, the Dean, and the Board, as well as meetings attended by representatives from each faction of the faculty to have deliberations and consider possible compromises. The attorney also insists that he is not an arbitrator or mediator because no litigation over the dispute is pending or even contemplated at this point. Is it proper for the attorney to serve in this capacity?

A.Yes, because a lawyer can serve as a third-party neutral when the lawyer assists two or more persons who are not clients of the lawyer to reach a resolution of a dispute or other matter that has arisen between them.

QUESTION 4. The big spender. An attorney pays $1,000 per month for a billboard advertisement for his firm, $2,000 per month for a few radio commercials, $3,000 per month for Internet advertising, and $4,000 per month for newspaper and magazine advertisements. The total amount for advertising is $10,000. At the same time, the attorney's average total income from legal fees is $15,000 per month. Is it permissible for the attorney to spend such sums on advertising?

A.Yes, because a lawyer may advertise services through written, recorded, or electronic communication, including public media, and may pay the reasonable costs of such advertisements or communications.

QUESTION 1. Across state lines. An attorney represented a seller in a business transaction involving industrial equipment. When the deal was complete, the purchaser sent the attorney a check for $7,000, the agreed-upon purchase price, with a letter directing the attorney to forward the money to his client (the seller). The attorney notified his client immediately that the check had come in. The client was traveling at the time, and asked the attorney to hold the funds until he returned from his trip. The attorney had only recently moved to this jurisdiction and opened a new firm, did not yet have a client trust account at any banks in the area, so he deposited the check in the client trust account in the neighboring state, where he had practiced until recently. He told the client that the funds would be in a separate client trust account, and explained that it would be out of state, and the client consented. As soon as the check cleared, the attorney wrote a check to the client for the full amount from the client trust account, which the client picked up in person. Did the attorney act properly in this case?

A.Yes, because a lawyer may deposit client funds in an out-of-state client trust account if the client gives informed consent to this arrangement.

QUESTION 6. Disinheriting the daughter. An attorney agreed to prepare a will for a client, a wealthy widow with three grown children. An earlier will divided her estate equally between her children, but the client now wants to modify the will to disinherit her only daughter, who disobeyed the client's wishes by marrying outside their nationality. The daughter is also a lawyer and is married to a lawyer, and the estate is substantial. The client's two sons are both working as manual laborers and they struggle financially. In the past, there had been some tension between the brothers and their sister, although the relationships seem to be cordial now. The attorney believes that disinheriting the daughter will ensure that the daughter and her husband will contest the will after the client's death and will rupture the tenuous relationship between the siblings. The client did not ask for the attorney's advice about disinheriting the daughter, she just insisted on it. The attorney initiated a debate about it, explaining that he believed it could be against the client's best interests and would cause unnecessary acrimony between her children. Was it proper for the attorney to initiate such advice when the client did not ask for it?

A.Yes, because a lawyer may offer unsolicited advice to a client when doing so appears to be in the client's interest.

QUESTION 5. All in the family. An attorney calls his friend, a close personal acquaintance, who was recently arrested for driving while intoxicated. The attorney advises that he saw the friend's arrest on the local police news and offers to represent his friend for the attorney's usual fee for handling such cases. The friend hires the attorney to represent him on the case. Are the attorney's actions proper?

A.Yes, because attorneys can solicit professional employment from family members, close personal friends, and persons with whom the attorney had a previous professional relationship.

QUESTION 3. Don't lecture me. An attorney represents a client, who lost his criminal appeals and is now serving a life sentence in a federal penitentiary. The client confesses to the attorney that he (the client) committed a murder for which a jury incorrectly convicted another (innocent) man. The client says he is happy that someone else took the fall for that crime and that he will never tell anyone. The attorney lectures the client about the morality of this situation, allowing an innocent man to face life imprisonment or even capital punishment for a crime that the client committed, and pleads with the client to reveal the truth. Was it proper for the attorney to bring morality into his consultation with the client, and to sermonize on this point for a few moments?

A.Yes, because in rendering advice, a lawyer may refer not only to law but to other considerations such as moral factors.

QUESTION 4. Money laundering. An attorney knew about another lawyer's involvement in an illegal money laundering enterprise, although the money laundering was unrelated to the other lawyer's law practice or representation of clients. Eventually, when federal law enforcement officials bring criminal charges against the other lawyer, who is part of another firm, the first attorney's awareness of the situation becomes evident. Could the attorney who knew of the wrongdoing and ignored it be subject to discipline?

A.Yes, because it is a violation of the Rules of Professional Conduct to fail to report serious fraud or criminal activity by another lawyer.

QUESTION 7. The victim's story. An attorney represented a client in a misdemeanor criminal matter involving minor vandalism. The attorney interviewed the victim, who incurred the property damage, hoping to learn more about the value of the damage and how frequently vandalism occurs in that neighborhood. The property owner explained to the attorney that the client had been demanding "protection money" from him and other business owners in the neighborhood for a long time, and that the vandalism followed his refusal to continue paying the protection money. The amount involved was substantial, and the attorney realized that the client could face much more serious charges for extortion. The attorney never discussed this with the client, and the client gladly accepted a plea bargain offer for a few months' probation on the misdemeanor vandalism charge. Several years later, the client died in a car accident, and the property owner became a business-world celebrity when he published a book about how businesses transform neighborhoods. A reporter eventually found the attorney and interviewed him about the vandalism incident, several years prior, that had damaged the property owner's building at the time. The attorney explained that the incident was in fact part of a larger extortion operation and that the business owner had handled the matter nobly. Should the attorney be subject to discipline for this disclosure?

A.Yes, because the confidentiality rule applies not only to matters communicated in confidence by the client but also to all information relating to the representation, whatever its source.

QUESTION 22. Living across the state line. An attorney lives in the border town of Nashua, New Hampshire, which is a forty-five-minute drive from Boston, Massachusetts. The attorney took the Massachusetts bar exam and passed it, and now seeks admission to the bar in that state, as she has a job offer from a firm in Boston and plans to commute there every day from her home in New Hampshire. The state bar of Massachusetts has a rule that lawyers must be residents of the state to obtain a license to practice law there on a regular basis, so it declines her application to the bar. When the attorney challenges this decision in federal court, will she prevail?

A.Yes, because the residency requirement violates the Privileges and Immunities Clause of the U.S. Constitution.

QUESTION 1. The former employee. In anticipation of trial, a plaintiff's attorney contacts several former employees of the defendant corporation and interviews them about the day-to-day operations of the company and the chain of command for addressing complaints. The attorney does this without permission from the defendant's lawyer. Was this proper?

A.Yes, consent of the organization's lawyer is not a requirement for communication with a former constituent of the organization that is a represented opposing party.

QUESTION 13. The law student intern. An attorney works for a mid-size law firm that employs two or three law students every year as summer associates. The manager of the student associates assigns one of them to work on the attorney's pending antitrust case, in addition to assignments for other lawyers at the firm. While researching an important issue in the case, the summer associate discovered an older Supreme Court decision that was unfavorable to their client. The summer associate decided not to tell anyone about the case, as the opposing party seemed to have overlooked it in their briefs. The attorney was not aware of any of this. The hearing went well for their side, and the associate never said a word about "burying" that Supreme Court case he had found. After winning at the hearing, the attorney complimented the summer associate for his fine work and rewarded him by treating him to lunch at an expensive restaurant. The judge's clerks, however, found the case, and the judge queried the lawyers about how they could have missed it. Opposing counsel admitted he had been negligent in doing legal research on the matter, and the summer associate then recounted how he hid the case from the attorney. Is the attorney now subject to discipline for what the summer associate did?

B.No, because the attorney did not know about the associate's conduct at the time it occurred, or while submitting briefs, or even when the hearing began.

QUESTION 1. Soliciting campaign contributions. As part of its CJC, a state has adopted a rule prohibiting judicial candidates from personally soliciting campaign contributions. Disregarding this rule, a candidate running for a judicial seat in that state signed and sent a mass-mailing requesting that the recipients give financial support to her campaign. She also posted the letter online. The state bar brought a disciplinary action against the candidate for violating the no-solicitation rule. The candidate responded by claiming that the prohibition violated her First Amendment rights to free speech. When she seeks judicial review of her claim, how should the court rule?

B.The court will hold that the First Amendment allows the state to prohibit in-person solicitations by judges as well as campaign fund-raising letters.

QUESTION 2. My old friend, the opposing party. An attorney represents the defendant in litigation over a car accident. The plaintiff, who was driving the other car, was a childhood friend and neighbor of the attorney—they keep in touch. As the defendant's attorney has known the plaintiff since childhood, he calls the plaintiff, who has retained counsel as well, to see if they can resolve the case without going to trial. Is the attorney subject to discipline for calling his lifelong friend?

B.Yes, as a lawyer shall not communicate about the subject of the representation with a person the lawyer knows to be represented by another lawyer in the matter, unless the lawyer has the consent of the other lawyer or is authorized to do so by law or a court order.

QUESTION 6. Tie us over for the weekend. A client retains an attorney to handle a criminal matter. The client delivers a retainer check to the attorney on Friday afternoon. The retainer check will only cover the work the attorney anticipates he will begin and complete the following Monday. Because the following Monday is a banking holiday, if the attorney deposits the retainer check into his client trust account on Friday afternoon, he will not have access to the funds until Tuesday. The attorney deposits the retainer check into his business checking account and pays himself on Friday before the firm closes with those funds. Is the attorney subject to discipline?

B.Yes, attorneys shall deposit amounts paid in advance into a client trust account and the attorney shall not withdraw the funds until fees are earned or expenses are incurred.

QUESTION 14. Profit sharing for paralegals. An attorney practices personal injury law, representing plaintiffs on a contingent fee basis. The attorney employs a paralegal to assist with preparing documents for litigation. The paralegal's salary arrangement is 10 percent of the firm's total net revenue each year. In years when the attorney wins several large cases, the paralegal receives higher wages, and in years when the attorney has no big wins, the paralegal receives almost nothing. The paralegal does not bring clients to the firm and does not participate in judgments about which clients to represent, or about how to handle the cases. Is the attorney subject to discipline for this arrangement?

B.No, because non-lawyers may participate in a firm compensation plan based on overall profit sharing.

QUESTION 20. The people want to know. An attorney represented a large corporation as a defendant in a toxic tort action. The matter had received little media attention and the corporate officers who retained the attorney emphasized the need to be discreet as long as possible, so that the pending litigation would have a minimal effect on stock prices. The representation necessitated that the attorney interview some of the employees involved in the incident that gave rise to the litigation, including some of the lowest-level unskilled laborers. A few of these individuals, as well as their co-workers whom the lawyer did not interview, asked the lawyer for details about what was happening with the case. The lawyer felt that they had a right to know about the case as it could affect the company, and their jobs, so he explained who the plaintiffs were, how strong the evidence appeared to be on each side, and the potential liability the company was facing. Could the attorney be subject to discipline for sharing this information with the company employees?

B.Yes, because a lawyer may not disclose to company employees any information relating to the representation except for disclosures explicitly or impliedly authorized by the organizational client in order to carry out the representation.

QUESTION 13. Too much information. In anticipation of a round of settlement negotiations over a business partnership breakup, a client authorized his lawyer to disclose that the client was having personal financial troubles, but added that the lawyer should "leave it at that—don't elaborate too much." The lawyer was to attend the settlement conference without the client. At the settlement conference, when the other parties pressed the lawyer about why his client seemed so inflexible about a settlement amount for dissolving the partnership, the lawyer said that his client was having personal financial problems. Counsel for one of the other partners asked, "Like what? Perhaps the other partners could do something to help, and it would make it easier to resolve the partnership breakup." The lawyer then explained that everyone in the room must keep the following information completely confidential and went on to explain that his client was on the verge of bankruptcy due to a gambling problem. He also explained, in a hushed tone, that the client had even assigned his equity share in the partnership to a business rival of the partnership to pay off a personal loan. The others were shocked, with a mixture of sympathy for their partner's gambling problem, and alarm at the implications of their main market rival owning a significant share of the existing partnership. One of the other attorneys, however, checked the terms of the original partnership agreement, and informed the rest that equity interests in the company were unassignable without a majority vote of the other partners, making the assignment legally void. Could the lawyer be subject to discipline for the disclosures he made at the settlement conference?

B.Yes, because he went beyond what the client had authorized him to disclose, thus breaching his duty of confidentiality to the client.

QUESTION 2. A clean start in another state. An attorney obtained admission to the bar in New York and practiced there for two years. She worked for Big Firm, which has offices in five states and a few locations overseas. After her two years in the New York office, the firm transferred her to its office in San Diego, California. The attorney then applied for admission to the California bar under a reciprocity arrangement, and the state bar admitted her without making her re-take the bar exam. After practicing in California for three years, somehow the New York state bar learned that the attorney had made false statements on her original bar application about misdemeanor arrests during college. The New York bar informed the California state disciplinary authority about this problem, and the California state bar commenced disciplinary proceedings against the attorney in California. Can the attorney be subject to discipline in California for false statements made on a bar application in another state?

B.Yes, because if a person makes a material false statement in connection with an application for admission, it may be relevant in a subsequent admission application or disciplinary proceeding elsewhere.

QUESTION 19. Corporate counsel and the tortfeasor employee. An attorney represents a corporation. One of the corporation's delivery trucks, driven by a corporation employee, had a tragic accident with a school bus full of children, and many children died. The delivery truck driver suffered severe injuries, but survived, and spent three weeks recovering in the hospital. In preparation for the wrongful death lawsuits by the deceased children's families, the corporation's attorney visited the truck driver in the hospital and interviewed him about the accident. The attorney did not explain that he was not representing the driver, or that the driver should retain his own lawyer. The unsophisticated driver may have assumed that his employer's lawyer was also looking out for his (the driver's) interests. The driver made some incriminating admissions to the lawyer about being slightly intoxicated at the time of the accident and having been careless while driving. He also admitted that at the time of the accident, he had taken the corporate delivery truck off its assigned route to attend to some personal business for about twenty minutes. Could the attorney be subject to discipline in this case?

B.Yes, because in dealing with an organization's employees, the lawyer should explain the identity of the client when the lawyer should reasonably know that the organization's interests are adverse to those of the employee with whom the lawyer is dealing.

QUESTION 3. Honest ads. An attorney advertised in a local newspaper. His advertisement reads, "I never charge more than $200 per hour for any type of legal work, and for simple legal problems such as uncontested divorces or name changes, I charge even less." The attorney once had a particularly complicated, tedious case in another jurisdiction for which he charged $250 per hour, but he does not expect such a case to arise in the future, though his fee would be higher if it did. The attorney's advertisement fails to state that some other lawyers in the community charge much lower fees. The advertisement includes a pencil drawing of an unrealistically handsome, but generic-looking judge sitting behind the bench in a courtroom with a gavel in his hand. Could the attorney be subject to discipline for this advertisement?

B.Yes, because it is not true that he never charges more than $200 per hour.

QUESTION 2. Third-party billing. An attorney uses an outside billing service to track client billing and to send bills to clients each month. The attorney submits computerized timesheets to the billing company at the end of each workday showing how much time he spent on which tasks for which clients. The billing company calculates the monthly totals and sends detailed bills to clients on the attorney's behalf. The clients are not aware that the attorney uses an outside billing service until they receive their bills. Has the attorney violated his ethical duties to his clients?

B.Yes, because submitting the client names, time worked, and tasks involved constitutes a disclosure of confidential information for which clients must provide informed consent beforehand.

QUESTION 6. Supporting the cause. An attorney is active within a new political movement and she has represented several members of the movement, who faced arrest or criminal charges for protesting and picketing. The attorney learns that police have arrested one of the prominent leaders of the movement for trespassing on private property during a protest, but that the movement leader is already out on bail. In response, the attorney calls the leader and offers to represent him in his case free of charge, explaining that she has experience representing other members of the movement in similar cases. The leader agrees to have the attorney represent him on a pro bono basis. The attorney wants to represent the leader because she admires him, but also because she believes it will generate terrific publicity for the firm's practice. Was it proper for the attorney to make this telephone solicitation?

B.Yes, because the attorney did not charge for providing these legal services.

QUESTION 17. Trouble with the Feds on the horizon. An attorney worked for a corporation as in-house counsel. The attorney discovered that the Chief Financial Officer falsified the corporation's quarterly earnings report to prop up the firm's share price, as the CFO's compensation is partly in stock options. The attorney knows that these misrepresented earnings appeared in the filings to the Securities and Exchange Commission and will eventually result in severe regulatory fines or civil liability for the corporation. The attorney thus reasonably believes that the violation is likely to result in substantial injury to the organization. The attorney confronted the CFO, but this proved unfruitful, and then he proceeded up the corporate chain of command, eventually going to the Chief Executive Officer and the Board of Directors. The officers and directors refused to address the problem because they thought it would send their stock prices into a freefall and make the corporation vulnerable to a hostile takeover from corporate raiders. Would it now be proper for the attorney to become a whistleblower and reveal the problem to the relevant government authorities?

B.Yes, because the attorney has exhausted all other reasonable avenues to address the problem internally, so the lawyer may reveal information relating to the representation whether Rule 1.6 permits such disclosure or not.

QUESTION 2. The buffer. An attorney has a busy transactional practice and frequently must handle client funds, either for making commercial purchases, sales, leases, dispute settlements, or other transfers. The attorney faithfully deposits client money in a separate trust account and does not commingle the funds with his own, except that he deposits enough of his own money in the account to cover the monthly bank service charges. He also put $1,000 in the account when he opened it and left it there, as a buffer in case there were any accounting errors, so the clients would never experience inconvenience due to the account being inadvertently overdrawn. The attorney keeps complete, accurate records of all deposits and withdrawals for seven full years, after which he destroys the records to preserve client confidentiality. Is the attorney acting improperly?

B.Yes, because the attorney should not have deposited the $1,000 buffer from his own funds in the account.

QUESTION 18. The disbarred lawyer-agent. A firm specializing in sports law represented several professional athletes as clients. The state disciplinary authorities suspended and eventually disbarred one of the associates at the firm, but the disciplinary action did not implicate the rest of the firm (the lawyer's misconduct had occurred completely outside the scope of his duties there). The firm retained the disbarred lawyer as a sports agent for some of the athletes who were clients of the firm. The disbarred lawyer would draft contracts for the athletes and negotiate deals for the firm's clients with their sports teams or with companies seeking the athlete's product endorsement. Could the partners at the firm be subject to discipline for facilitating the disbarred lawyer in the unauthorized practice of law?

B.Yes, because the disbarred lawyer is engaging in the unauthorized practice of law by drafting and negotiating contracts for the firm's clients.

QUESTION 9. Stories over drinks with friends. An attorney represented a client, who was a defendant in a criminal prosecution. The client's trial ended in a conviction and a life sentence. After all possible appeals were complete, the attorney's representation of the client ended. The attorney sent the client a letter, which the client received in prison, explaining that his representation was now ending and providing a detailed accounting of all billing matters. No outstanding bills remained. Several years later, the attorney met with some former law school classmates at an alumni event, and they swapped stories over drinks about some of their cases over the years. The attorney mentioned the client, but only by first name, and explained how the guilty verdict felt like a failure on his part even though he knew the client was guilty because the client's friends and family members had all witnessed the crime and told the attorney privately what they had seen. Could the attorney be subject to discipline for disclosing confidential client information?

B.Yes, because the duty of confidentiality continues after the client-lawyer relationship has terminated.

QUESTION 11. Seller beware. A client intends to purchase a parcel of real estate and retained an attorney to analyze the seller's title to the property. The attorney requests information from the seller regarding the seller's original acquisition of the property and obtains additional information from the local tax assessors and title registry. The attorney concludes that the seller does not have clear title to the property and informs the seller of this opinion when the seller asks him about it. The seller forbids the attorney to disclose the information to the prospective purchaser of the property and insists that he showed the attorney his documents about the original acquisition of the parcel with the understanding that the attorney would not say anything unfavorable. May the attorney inform the prospective purchaser of his opinion about the title?

B.Yes, because the seller does not have a client-lawyer relationship with the attorney.

QUESTION 3. Accusing the Judges. A district attorney in a small city was on a losing streak, and she decided it must be the local judges who were at fault. She used her social media accounts to attack some of judges before whom she would regularly appear, for refusing to order reimbursement of the district attorney's costs of investigating cases and bringing them to trial. She also criticized the local judiciary for the longstanding backlog of criminal cases awaiting trial. At one point she even alleged that some local judges had connections with a drug cartel. These were all unsubstantiated claims, and the accusations were false. The named judges did not in fact have backlogs on their own dockets, and no one from the district attorney's office had submitted any formal requests for reimbursement of investigatory costs. The district attorney eventually faced disciplinary proceedings over this social media campaign against the judges, and her defense was that such statements had First Amendment protection and reflected her personal beliefs. Should the district attorney be subject to discipline for her public statements?

B.Yes, because the statements were false, and the district attorney did not have an objectively reasonable belief, at least for an attorney, that these statements were accurate and true.

QUESTION 12. Just go with it. An attorney represents a client, who wants to sell his business. A prospective purchaser has required from the client an evaluation of the business' solvency, detailing its current liabilities, potential liabilities, revenue, and assets. The client provides the attorney with documents pertaining to each of these issues, and explains to the attorney in confidence that he has often understated the earnings of the business to avoid paying taxes on the business profits. Now he is concerned that the prospective purchaser will undervalue the profitability of the business and refuse to pay an appropriate price to purchase it. He asks the attorney to adjust the earnings figures upward by 25 percent, the same amount by which the client falsely lowered them in the corporate records, to portray the business accurately to the potential purchaser. The attorney finds this objectionable and prepares a report based on what the records say regarding the earnings and gives the evaluation directly to the purchaser. When the client learns about this, he explains to the prospective purchaser over the phone what happened. Despite the low reported earnings, the purchaser pays the client's asking price for the business, because of the client's truthful representations over the phone. Could the attorney be subject to discipline for his conduct in this matter?

B.Yes, because under no circumstances is the lawyer permitted to knowingly make a false statement of material fact or law in providing an evaluation for a third party.

QUESTION 8. Fighting over the fees. A client met with an attorney for a free consultation, and explained that she had met with two other lawyers for consultations and that she planned to hire one of the three to provide the legal services necessary to set up her professional business. The attorney needed to make a good impression on the client, so he mentioned a few prominent accountants and physicians in town whom the attorney had represented and helped with incorporating their partnerships or practice groups. These former clients had never explicitly authorized the attorney to disclose his representation of them in these matters. The client hired the attorney, and the attorney provided the legal services necessary to set up her business. Unfortunately, a dispute arose between the client and the attorney over the fees, and this fee dispute turned into litigation between the attorney and the client. In order to support his claims and defenses in the fee dispute, the attorney had to disclose to the tribunal exactly what he did for the client and the complexity of the issues involved, which necessarily involved the disclosure of confidential information. Was it proper for the attorney to disclose this confidential information about the client merely to prevail in a fee dispute?

B.Yes, the lawyer may reveal information relating to the representation to establish a claim in a fee dispute between the lawyer and the client.

QUESTION 9. I won't do it. On his website, an attorney explains that he handles most areas of personal injury law, and then displays in large, bold letters: "I DO NOT REPRESENT CLIENTS IN CRIMINAL MATTERS OR DIVORCE MATTERS—PLEASE FIND ANOTHER LAWYER IF YOU ARE FACING CRIMINAL CHARGES OR NEED TO LEAVE YOUR SPOUSE." Is it improper for a lawyer to make such a statement in his website or advertising materials?

C.No, because a lawyer may communicate the fact that the lawyer does or does not practice in particular fields of law.

QUESTION 6. Under orders. An attorney represents a client before an Administrative Law Judge in a regulatory enforcement matter. The Administrative Law Judge orders the attorney to disclose whether the client was informed by counsel about the regulatory requirements in question before the violation occurred. The client forbids the attorney to answer the question. The attorney initially objects, but the Administrative Law Judge insists. Could the attorney be subject to discipline for disclosing such confidential client information to the Administrative Law Judge?

C.No, because a lawyer may comply with an order to reveal information relating to the representation of a client by a court or by another tribunal or government entity claiming authority pursuant to other law to compel the disclosure.

QUESTION 1. Emergency room patients. An attorney is dating a woman whose sister works as a nurse in a hospital emergency room. The attorney gives the nurse, his girlfriend's sister, a stack of his business cards and law firm brochures, and offers to pay her $200 for any clients who hire him because of her referrals, with the understanding that she will not refer patients to any other lawyers. The nurse recommends several patients per month to the attorney for representation in personal injury claims, and one or two per month actually hire the attorney to represent them. Is such an arrangement proper?

C.No, because a lawyer shall not give anything of value to a person for recommending the lawyer's services, with certain exceptions not applicable here.

QUESTION 5. Just a hunch. An attorney suspects that another lawyer in his firm has violated the Rules of Professional Conduct in a rather serious matter, but has no first-hand knowledge of the situation—his suspicion rests on the fact that the other lawyer seems to be acting paranoid and evasive, and a number of strange coincidences have occurred in his cases. Does the attorney who suspects something seriously wrong is afoot have a duty to report the other lawyer to the state bar disciplinary authority?

C.No, because he does not have actual knowledge of the violation.

QUESTION 3. Discussing a case with the partners. An attorney is a partner in a seven-lawyer firm. A client retained the attorney to handle his workers' compensation matter. The attorney did discuss with the client that he would normally disclose to the other partners in the firm some of the details about his cases and clients, and the client expressly forbade the attorney from telling anyone in his firm anything about his case. Nevertheless, at the weekly meeting of the partners, as everyone discussed their pending cases, the attorney explained the client's case and solicited input from the partners. One partner had an ingenious suggestion that was helpful to the client's case. The attorney mentioned to the client in their next phone call that one of his partners had suggested a strategy that could turn the case in the client's favor. The client was upset that the attorney had discussed the case with anyone else. Was it proper for the attorney to discuss the case with the others at the firm?

C.No, because lawyers in a firm may not disclose to each other information relating to a client of the firm if the client has instructed that particular information be confined to specified lawyers.

QUESTION 7. After the storm. After a hurricane damaged hundreds of homes in a southeastern state, an attorney, who practices in that state, sent letters to a dozen homeowners in the affected area offering to represent them in their insurance claims arising out of the storm damage. Each letter was handwritten and personalized, and the attorney addressed each envelope by hand so that recipients would perceive it as a personal letter and would be more likely to open it and read it. Nothing in the letters was coercive or threatening. Were the attorney's actions improper?

C.No, because such communications can be mailed to inform the public about the need for legal services, and about the qualifications of available lawyers and law firms, without subjecting the public to live person-to-person persuasion that may overwhelm a person's judgment.

QUESTION 8. New work, old clients. An attorney specializes in employment law, especially employer-provided benefits, as well as healthcare law. After Congress passes sweeping legislative reforms for the regulation of employer-sponsored healthcare plans, the attorney called her former business clients offering to help them sort through the changes in employee benefit plans that the new laws would require. These individuals had not initiated the contact, nor had they indicated that they did not wanted to be contacted by the attorney. Could the attorney be subject to discipline for calling these former business clients?

C.No, because the attorney called only to former clients.

QUESTION 21. Exclusively federal practice. An attorney has a firm in a state in which the attorney lacks a license to practice law. The attorney's legal work, however, consists entirely of representing local inventors before the United States Patent and Trademark Office in Washington, D.C., either by correspondence or by traveling to appear there in patent proceedings. A relevant federal statute states that non-lawyers may represent patent applicants before the USPTO. The attorney does no other legal work for clients—if clients need representation for family law matters, employment matters, incorporating businesses, or personal injury suits, the attorney refers them to outside counsel. All the attorney's clients, however, are located in the state where the firm has its office, and the attorney is unlicensed there. Is the attorney subject to discipline?

C.No, because the attorney is providing services authorized by federal law, which preempts state licensing requirements.

QUESTION 5. Asking for advice. An attorney is representing a client who is a celebrity. The attorney is confused about whether he may publicly disclose information that he learned in confidence from his client if the information is already a matter of public record, and his research indicates there is a split of authority on this question. The attorney calls another lawyer who specializes in lawyer malpractice and disciplinary matters to seek advice. The other lawyer agrees to provide an opinion and to keep the conversation a secret. The attorney tries to use a hypothetical to explain the problem, but given the client's national reputation and celebrity status, the other lawyer knows immediately who the client is, and can easily surmise the nature of the confidential information. Is the attorney subject to discipline for disclosing confidential information about his client?

C.No, a lawyer may reveal information relating to the representation of a client to the extent the lawyer reasonably believes necessary to secure legal advice about the lawyer's compliance with the Rules of Professional Conduct, even without client authorization.

QUESTION 2. Don't worry. . . . A client repeatedly calls an attorney to discuss her pending divorce case. The client wants above-guideline child support, alimony, and a large percentage of the estate, even though the parties have only been married two years. The attorney has continuously given his honest opinion about what he believes the client is eligible to receive, and what he believes she may receive in the divorce based on his experience. The client has recently become angry with the attorney because she is unhappy with his opinion. She has even asked, "Are you working for me or my husband?" In an effort to keep the client happy, the attorney begins to tell the client what he believes she is eligible to receive when she asks, but simply states "the court will decide" when the client asks the attorney what he believes she will receive. Are the attorney's actions proper?

C.No, an attorney should give his honest opinion about the case when asked, even if the opinion is unsatisfactory to the client.

QUESTION 13. The former mediator. An attorney, who often serves as a court-appointed mediator, received an appointment to mediate the divorce case between a husband and wife. The case settled in mediation and the divorce became final soon after. A year later, the husband sought to retain the attorney to represent him in a modification suit against his ex-wife. The attorney accepted the case and sent a letter to the ex-wife advising her that her ex-husband had retained him to represent him in a modification suit. Are the attorney's actions proper?

C.No, an attorney who previously served as a third-party neutral is required to obtain informed consent, confirmed in writing, from all parties to the proceeding prior to representing a party in a suit connected to the previous matter.

QUESTION 16. Working with Washington lawyers. An attorney in a state that has adopted the Model Rules in their current form enters into a fee-sharing agreement with a lawyer admitted in Washington, D.C., which permits fee sharing with non-lawyers and multidisciplinary practices. They collaborate on a case and divide the fees as agreed. The attorney from the Model Rules state is aware that the other attorney will share his part of the fees with non-lawyers in the D.C. office; in fact, the D.C. lawyer's firm has accountants who hold an ownership share in that firm. Is the non-D.C. attorney subject to discipline for indirectly sharing legal fees with non-lawyers, given that he practices in a state that forbids fee sharing with non-lawyers?

C.No, as long as the first attorney shares fees only with another attorney, it does not matter if the other attorney shares fees with non-lawyers as permitted by his home jurisdiction.

QUESTION 9. Punished in both states. An attorney had a license to practice law in two jurisdictions—his home state where he lived and had his main office, and a neighboring state where he represented several clients each year. The attorney committed serious professional misconduct in his home state and received a public reprimand from the state disciplinary authorities. All the conduct took place in his home state, the client resided in the state, and the representation took place entirely within his home state. The lawyer's conduct would have violated the rules in either of the jurisdictions where he had a license to practice law, because it involved commingling client funds with his own money, and the states had nearly identical rules concerning this activity. After the attorney received a public reprimand in his home state, where the misconduct occurred, the state bar disciplinary authority in the neighboring state (where he also practiced) then commenced disciplinary proceedings against him as well. In the end, the neighboring state bar suspended his license for six months in that state, a much more severe sanction than the public reprimand he received in his home state, where the misconduct actually occurred. The attorney claims that the neighboring state bar has no jurisdiction over conduct that occurred entirely outside of the state. He also objects that the second punishment raises double jeopardy concerns. Is the attorney correct?

C.No, because a lawyer may be subject to the disciplinary authority of two jurisdictions for the same conduct, and may receive different sanctions in each state.

QUESTION 3. When the client disputes the fee. An attorney represented a client in a contention litigation matter, at the end of which the attorney received a settlement check for an agreed-upon amount from the opposing party ($100,000). The client had agreed to the amount but was unsatisfied and blamed the lawyer for the disappointing settlement amount. The attorney called the client to inform her that the check had arrived, and explained that he would forward the amount minus his fees and the expenses, which constituted half of the amount (the jury consultants and experts in the case had turned out to be very expensive). The client was furious and said that the expenses should have been included in the attorney's contingent fee, and that the attorney was not entitled to the original contingent fee in any case because the case had never gone to a verdict and had settled for a mediocre amount. Pursuant to their retainer agreement, the client and attorney agreed to schedule arbitration over the disputed fees and expenses as soon as possible, which realistically would be three or four months later. In the meantime, the attorney kept the money in the client trust account until they could resolve the dispute. Was this proper?

C.No, because the attorney should have paid the client $50,000 immediately and held only the remainder until the dispute was resolved.

QUESTION 1. What's in a name? A government entity provides grants to a legal aid office that represents indigent individuals. The government entity requires reporting of the names of clients, brief factual summaries, and the type of representation involved for all matters where the government entity's funds provided the financial support for the representation. This information helps ensure that the funding is going to its intended purposes. An attorney works for the legal aid office. Most of his clients are uneducated and unsophisticated, so he does not explain to them that he must disclose their information. Is it proper for the attorney to make the required disclosures to the funding agency without obtaining the clients' informed consent?

C.No, because the client names, basic facts, and types of cases are confidential information, and require client authorization for disclosure.

QUESTION 2. The major campaign contributor. In a state with elected judges, a certain candidate (the challenger) for the state supreme court sought to unseat an incumbent justice. The president of a coal company in that state set up a nonprofit organization to advocate for the removal of the incumbent justice, and contributed millions of dollars to this entity, which ran an extensive advertising campaign criticizing the incumbent justice. Even though the president did not give this money directly to the challenger, the campaign was effective, and the challenger won the election and became a state supreme court justice. The millions contributed to the nonprofit by the coal company president exceeded the total amount of all other campaign contributions in that election. Shortly thereafter, a lawsuit involving the coal company, which had been working its way through the courts, came up for review by that state supreme court. The justice who had won the election joined a 3-2 majority ruling in favor of the coal company in a case worth more than $50 million. The other party had petitioned for recusal by the newly seated justice, given that the coal company's executive had contributed millions of dollars to help the justice win the election, but the justice did not recuse himself, explaining in a lengthy memorandum why he thought he could render an unbiased decision in the case. Was it proper for the new justice to participate in the decision in this case?

C.No, because the coal company's disproportionate campaign contributions created an impermissible appearance of bias, even if the justice was in fact unbiased.

QUESTION 7. Whether to appeal. An attorney represented a client in tort litigation against a pharmaceutical company over injuries allegedly resulting from one of the company's drugs. During a pretrial hearing about the admissibility of certain evidence, the court ruled against the attorney and ordered that the evidence was inadmissible at trial. The attorney then contacted a reporter from a prominent newspaper and gave him a lengthy interview explaining the case, discussing the upcoming trial, and giving the reporter the very evidence that the court had held should be inadmissible at the trial. The newspaper ran the story on the same day that jury selection began for the trial. Opposing counsel moved to disqualify the attorney due to misconduct in the matter, that is, the public disclosure of the inadmissible material in an attempt to taint the jury pool. The court agreed to disqualify the attorney on the eve of the trial. Another firm was already representing the client as co-counsel, so that firm agreed to continue with the trial work alone. The attorney filed an interlocutory appeal, which he lost at the appellate court and appealed to the Supreme Court. Delaying the trial with this interlocutory appeal was clearly against the client's interest, but it was necessary for the attorney to continue to handle this big case. Is it proper for the attorney to recommend to the client that they appeal his disqualification, if it is not clearly in the client's interest to do so?

C.No, because the decision to appeal should turn entirely on the client's interest.

QUESTION 12. The letter to the physician. An attorney represents Conglomerate Corporation. One of the company's employees died in a bizarre accident at the worksite, and Conglomerate asked the attorney to investigate the cause and the company's potential liability. As part of her investigation, the attorney wrote a letter to a physician he knew personally. The attorney's letter set out the circumstances of the employee's death and requested the physician's professional opinion as to the cause. The letter also explained that the attorney was preparing for a "possible claim" by the employee's estate for worker-compensation benefits, and potential wrongful death damages. When litigation ensued, the deceased's estate later requested a copy of the letter that the attorney sent to the physician. Must the attorney disclose the letter?

C.No, because the letter is non-discoverable work product.

QUESTION 22. The dreaded conversation. An attorney served as in-house counsel for a corporation, and uncovered illegal actions taken by a particular senior manager (not the Chief Executive Officer or any comparable officer or director, but an individual with decision-making authority and several direct subordinates in the organization). The senior manager had a reputation for being arrogant and unreasonable, and he and the attorney had clashed on several occasions and were barely on speaking terms. At the same time, the senior manager was exceptional in his area of expertise and was an asset to the company despite his unpleasant demeanor. The attorney summoned the nerve to confront the senior manager about the problem as graciously as possible, and the senior manager's initial response was to be dismissive, saying that he was unaware of any laws or regulations that he might have violated. The attorney walked away from the conversation discouraged and planned to take the matter up with the corporate officers, and potentially the Board of Directors. Before doing so, he reconsidered and returned to the manager, and patiently explained to him the relevant laws and regulations that the manager had violated. The senior manager begrudgingly accepted the attorney's advice and took all necessary measures to rectify the wrongdoing and prevent any long-term repercussions. The senior manager also insulted the attorney, called him incompetent for not bringing up the matter earlier, and suggested that the attorney's incompetence was due to the attorney's ethnic background. Could the attorney be subject to discipline for not referring the matter of the illegal actions to a higher authority in the corporation?

C.No, because the manager took the lawyer's advice.

QUESTION 9. What's missing here? An attorney represents a client before a government agency that enforces securities regulations. As part of the representation, the attorney must prepare an opinion concerning the legality of the securities registered for sale under the securities laws, for submission to the government agency, which requires such reporting. The client authorizes the attorney to prepare the written opinion but insists that the attorney exclude any mention of a particular business loss the client's company incurred recently, in order to avoid upsetting the shareholders. To preserve the client's confidential information, the attorney prepares the written opinion without the information the client asked him to withhold. The report does not mention that it excludes some unfavorable information. The attorney prepares the written opinion and gives it to the client, who submits it to the agency. Is it proper for the attorney to follow the client's instructions in preparing this report?

C.No, because when a lawyer's report categorically excludes certain issues or sources, then the lawyer must describe in the report any such limitations that are material to the evaluation in the report.

QUESTION 14. The good deal. A husband and wife are attending court-ordered mediation with an attorney, who is serving as the neutral mediator. The husband has retained counsel, but the wife has not. During mediation, the wife asks the mediator for his advice, and asks whether he believes that the husband's offer is a "good deal" for her. The attorney explains that his position as mediator only allows him to facilitate the negotiating process. The wife continues to seek the attorney's advice about the settlement proposals the husband makes. The attorney finally tells the wife what she is getting is a decent percentage of the estate and that he believes it to be a "good deal" for her. The attorney also informs the wife again that he does not represent her and that anything he says is merely general information, not legal advice. Are the attorney's actions proper?

C.No, the lawyer should decline to advise her, and instead explain more carefully the difference between the lawyer's role as a third-party neutral and a lawyer's role as one who represents a client.

QUESTION 3. Straight to the top. While preparing for a trial over workplace discrimination, the plaintiff's attorney contacts the owner and chief executive officer (CEO) of the defendant corporation and interviews her about the day-to-day operations of the company and the chain of command for addressing personnel complaints. The owner/CEO is not personally involved in the matter of the pending litigation—she actually never met the plaintiff warehouse worker who claims to be the victim of workplace discrimination, she is not on the witness list to testify at trial, and nobody has suggested that she was responsible for the wrongdoing. Even so, she has the power to settle the case or stipulate to a judgment amount, so the plaintiff's attorney talks to her directly. The attorney does this without permission from the corporation's lawyer, whom the company's general counsel hired; general counsel is an employee three steps below the CEO in the organizational chart. Was this communication by the plaintiff's attorney proper?

C.No, the rules prohibit a lawyer from communicating with the constituent of a represented organization (opposing party) who has authority to obligate the organization with respect to the matter.

QUESTION 21. Going up the chain of command. An attorney is in-house counsel for a large international corporation and has daily contact with higher-level executives and managers. One day, a senior executive mentions casually to the attorney that he has offered lucrative stock options, worth millions of dollars, to a foreign government official who has agreed to give the firm an exclusive contract to provide certain goods and services to the foreign state. The executive seems to think this is normal and good for the company, but the attorney believes it constitutes bribery of foreign officials, which would violate the Foreign Corrupt Practices Act, and could subject the company to enormous fines and penalties. The attorney explains her concerns to the executive, including her concern that he could face personal criminal charges in addition to bringing liability on the corporation, and she reminds him that she represents the corporation, not him personally. The executive is dismissive of her concerns, even though she approaches him several times about the matter. How must the attorney proceed?

C.She should approach the executive's immediate corporate superior, advising those next up the chain of authority to stop the transaction and take appropriate actions against the executive involved.

QUESTION 6. It starts with a phone call. A business owner hires an attorney hoping to enforce a non-compete agreement against a former employee at their technology firm. According to the client, a rumor started going around just this past week that the former employee either had started his own consulting practice nearby or was working for a nearby competitor. Each scenario, if true, could violate the non-compete agreement. The employee left the client's company on bad terms about three weeks ago. The client provides a copy of the non-compete agreement, and speculates that the former employee may have forgotten about the agreement (which he signed fifteen years ago), and may even be oblivious to the fact that he is violating it. The attorney decides that the first step is to call the former employee and ask him whether he has found another job yet or has started his own business. The attorney assumed that the former employee would not have retained counsel yet to challenge the non-compete agreement, given the client's comments about him, and the brief time since the events had unfolded. The former employee answers the phone, and after the attorney identifies himself as counsel for the business client, the former employee explains that he has started his own rival company, and that he believes the non-compete agreement is invalid under state law. When the attorney asks why it would be invalid, the former employee says that his own lawyer told him that recent changes in state law make the previous agreement void, and that they plan to challenge the agreement in court. The attorney asks him to have his own counsel contact him so that they can discuss potential settlement for the dispute. Has the attorney acted properly?

C.Yes, as the prohibition on communications with a represented person only applies in circumstances where the lawyer knows that the person is in fact represented in the matter to be discussed, and this means that the lawyer has actual knowledge of the fact of the representation.

QUESTION 7. Simple battery. A criminal court found that an attorney had engaged in domestic violence against his partner, and convicted the attorney of misdemeanor-level battery, for which he served a six-month term of probation. Could the attorney be subject to professional discipline as well?

C.Yes, because crimes of violence indicate a lack of the character traits required for law practice.

QUESTION 17. The marketing guru and his contract. The attorney hires a nationally known Internet-marketing specialist, a tech guru, to help develop the firm's reputation and attract new clients. The Internet specialist has made millions on previous tech startups, while the attorney is not well known and has been practicing for only two years. The tech guru demands certain terms in the contract that require the attorney to confer with the tech guru about accepting clients that were former clients of the tech guru, to avoid conflicts of interest. The attorney must also clear any litigation positions, approaches, or strategies that pertain to intellectual property or Internet marketing liability with the tech guru, to avoid positions that would jeopardize the guru's other business. Is the attorney subject to discipline for this arrangement?

C.Yes, because in this case, a non-lawyer has a contractual right to direct or control the professional judgment of the lawyer.

QUESTION 15. Profit sharing and votes for paralegals. An attorney practices personal injury law, representing plaintiffs on a contingent fee basis. The attorney employs a paralegal to assist with preparing documents for litigation. The paralegal's salary arrangement is 10 percent of the firm's total net revenue each year. In years when the attorney wins several large cases, the paralegal receives higher wages, and in years when the attorney has no big wins, the paralegal receives almost nothing. The paralegal does not bring clients to the firm, but does participate in judgments about which clients to represent, how to structure contingent fee arrangements, and how much to seek in damages after a verdict, as these matters directly affect the paralegal's income as well as the attorney's. Could the attorney be subject to discipline for this arrangement?

C.Yes, because this paralegal is sharing profits with the attorney, and is able to influence the professional judgment of the lawyer under this arrangement.

QUESTION 16. Clash of the titans. An attorney worked for a corporation as its in-house counsel. Hostility breaks out between the Chief Executive Officer (CEO) and the Chief Financial Officer (CFO), with each threatening to sue the other over allegations of slander, libel, trespass to chattel, and so on. Does this personal clash between top managers present the attorney with a conflict of interest?

D.No, because a lawyer employed by an organization represents the organization acting through its duly authorized constituents, so the lawyer represents neither of these officers individually.

QUESTION 5. Risky business. A client hires an attorney to help with the legal documents necessary to liquidate most of his investments so that he can use the cash to fund a new business venture. The client explains that he plans to quit his regular job and start a new career working from home as a "day trader," buying and selling stocks online every day in hopes of making large profits. The client has no experience or training in finance or investments, but he attended a seminar that featured testimonials from others who claimed to have made millions as day traders. The attorney thinks this is a foolish idea, but the client does not ask the attorney for his advice. Does the attorney have an ethical duty to caution the client against his seemingly reckless decision?

D.No, because a lawyer is not expected to give advice until asked by the client.

QUESTION 6. Mandatory reporting. An attorney works at a large firm and sees almost daily violations or potential violations of the Rules of Professional Conduct, though nearly all of them are minor and cause no harm or injury to the clients, third parties, or anyone else. For example, some lawyers represent co-defendants in cases where conflicts could arise at some point in the litigation, though the cases always seem to settle before any such scenarios develop. In other instances, certain lawyers seem to do minimal research on their cases or sometimes neglect client matters for weeks at a time, but again there has not been a case that was particularly serious. Does the attorney have a duty to report these violations to the state disciplinary authority?

D.No, because a lawyer must report only those offenses that a self-regulating profession must vigorously endeavor to prevent; if a lawyer were obliged to report every violation of the rules, the failure to report any violation would itself be a professional offense.

QUESTION 11. The campaign volunteer. An attorney volunteered for a judge's reelection campaign because he hoped to receive court appointments. He drives the judge from campaign stop to campaign stop without receiving any compensation for his time or effort. The judge wins re-election, and then shows his gratitude to the attorney by frequently appointing him to represent indigent defendants at the state's expense. The appointments turn out to be lucrative and to generate substantial fees for the attorney. Could the attorney be subject to discipline for soliciting funds for a judge with such self-interested motives?

D.No, because for purposes of this rule, the term "political contribution" does not include uncompensated services.

QUESTION 1. That was then, this is now. An attorney has been practicing for five years, but on her application to the bar five years earlier, she had stated that she had attended a particular private high school, when in fact she had attended a public high school. An unhappy client recently filed a grievance against the attorney, which was frivolous, but the state disciplinary authority had to conduct a routine, preliminary inquiry into the matter in order to make a determination that the complaint merited dismissal. The disciplinary board member assigned to the case had attended the elite private high school from which the attorney claimed to have graduated, and made a mental note of the attorney's high school when he did a cursory review of her bar admission files. He thought it was strange that he had never seen or heard her name at any alumni or reunion functions, as they had supposedly graduated the same year and the classes were small. On a hunch, the board member checked the alumni lists for the school and discovered that the attorney had lied on her application to the bar five years earlier. When asked about this issue, the attorney said she could not be subject to discipline now for the misstatement she made several years ago, and that the board lacked jurisdiction because it was unrelated to the current grievance complaint. Is she correct?

D.No, because if a person makes a material false statement in connection with an application for admission, it may be the basis for subsequent disciplinary action if the person is admitted.

QUESTION 1. Protecting the client's feelings. Halfway through a trial, an attorney can tell that his client is going to lose. The opposing party successfully impeached the attorney's only favorable witness, and the judge has already told the parties that he plans to follow the state's model jury instructions for this type of case, which effectively preclude the legal theory that the attorney had made the centerpiece of his case. During a lunchtime break, the client turns to the attorney and tearfully asks if they still have any chance of winning. The attorney does not want to make her cry and feels very awkward about the situation, so to spare her feelings, he assures the client that they still have a good chance of prevailing. The attorney is representing the client on a contingent fee basis, so he knows it will not cost the client any more in legal fees to finish the trial. At the same time, there is still an open settlement offer on the table from the other party, albeit a very small, unsatisfying settlement, which the client could accept at any time if she wants to terminate the litigation. Is it proper for the attorney to feign confidence to protect his client's feelings?

D.No, because in representing a client, a lawyer shall render candid advice.

QUESTION 4. Client dispute with a third-party creditor. An attorney received from the opposing party $150,000 as a settlement for the attorney's client. Before the attorney could disburse the funds to the client, a third-party judgment creditor with a court-ordered lien against the client contacts the lawyer demanding disgorgement of the client's funds to satisfy the amount of the judgment, from a matter in which the lawyer did not represent the client. The client instructs the attorney to give the money to the client immediately and not to give anything to the third-party judgment creditor. Preliminary inquiries suggest that the third-party judgment creditor has a valid court order to execute on the client's assets. The attorney did as the client instructed him to do, disbursing the funds promptly to the client, and informed the judgment creditor to take up the matter with the client directly. Did the attorney act properly?

D.No, because in this type of situation, the lawyer must refuse to surrender the property to the client until the claims are resolved.

QUESTION 20. Recent graduates awaiting their bar results. An attorney hires three new associates upon their graduation from law school in a neighboring state. The associates passed the bar in the neighboring state, but they are still unlicensed in the attorney's state. The associates confine their work to conducting research, reviewing documents, and attending meetings with witnesses in support of the attorney, who is responsible for all the litigation. The research done by the associates, however, is far beyond the capabilities of a paralegal or a typical law student associate. Is the attorney subject to discipline for this arrangement?

D.No, because the associates merely conduct delegated work under the attorney's supervision, for which the attorney is ultimately responsible.

QUESTION 18. Now that we're already in trouble. A large corporation was under investigation by a government regulatory agency over possible violations of securities law. The corporation hired an attorney to represent it in the matter and authorized the attorney to make a full internal investigation to discover the merits of the accusations. The attorney discovered that a high-level manager had falsified quarterly earnings reports, a clear violation of the law that could expose the corporation to devastating sanctions and civil liability. The attorney confronted the officer involved, but this proved unfruitful, and then he proceeded up the corporate chain of command, eventually going to the Chief Executive Officer and the Board of Directors. The officers and directors refused to address the problem because they thought it would send their stock prices into a freefall and make the corporation vulnerable to a hostile takeover from corporate raiders. Would it now be proper for the attorney to become a whistleblower and reveal the problem to the relevant government authorities?

D.No, because the attorney has a duty of confidentiality to the corporation, and the corporation hired the attorney to defend the organization against a claim arising out of an alleged violation of law.

QUESTION 11. Responsibility in hiring. A small firm employs several associates who work under the supervision of the partners, as well as three clerical staff. The most recently hired associate has a complicated situation with his license to practice law. The associate graduated from an accredited law school, successfully passed the state bar exam, and applied for admission to the bar, believing he had met all the eligibility requirements. He had no criminal record or history of academic misconduct, or any other problems meeting the traditional character and fitness requirements. The state bar approved his application and he attended his swearing-in ceremony. The state legislature, however, had recently passed a statute creating the option of a legislative veto for lawyers seeking admission to practice law in the state. The sponsors of the enactment had stated that its purpose was to prevent the grown children of illegal immigrants from becoming lawyers, even though the bar applicant might be a United States citizen "just because they happened to be born here." The associate was born in Arizona one month after his parents had moved there illegally from a country in Central America. A staff member of the relevant legislative committee flagged the associate's name from a list of recent bar licensees, along with three others in his situation. During a special session of the legislature, the state legislature exercised the equivalent of a legislative veto, narrowly passing a special act that permanently disbarred the associate and the others for the sole reason that their parents were illegal aliens. The associate received official notice of his disbarment from the Office of Legislative Counsel, not from the state bar. This occurred one week after the associate's swearing-in ceremony by the state bar, and two days after he started working at the firm. A notice of the disbarments appeared in the next issue of the state bar journal, but most of the firm was unaware of the situation, except for one managing partner in whom the associate had confided. Could the partners at the firm be subject to discipline for employing the associate as an attorney, despite challenging any such discipline in court?

D.No, because the courts have inherent power to regulate the legal profession, and the legislature's action could not survive a court challenge.

QUESTION 11. The internal inquiries by corporate counsel. Executives at a large pharmaceutical manufacturer discovered that one of the company's foreign subsidiaries had paid bribes to local government officials to obtain lucrative government contracts. During the internal corporate investigation of the matter, the corporation's directors asked their in-house General Counsel to send written inquiries to dozens of the corporation's foreign managers about whether similar payments or bribes were occurring elsewhere. After reviewing the responses to these inquiries and following up with phone calls and meetings, the corporate directors self-reported any questionable transactions to the Internal Revenue Service (IRS). The IRS then demanded to see all the original written responses to these internal inquiries. The corporate directors and General Counsel refused, claiming that the information was privileged. Should the government (the IRS) be able to compel production of these documents with a court subpoena?

D.No, because the information was provided at the request of the corporate directors to the in-house attorney, and the information was work-related, and was necessary for obtaining legal advice.

QUESTION 12. Certified long ago. An attorney entered into an exclusive reciprocal arrangement with a local advertiser, in which the lawyer agreed to advertise her firm solely through that advertising agency, for normal market rates, and the advertising agency agreed not to take any other law firms in the region as clients. The agency ran newspaper and billboard ads based on information supplied by the lawyer, which included a claim that the lawyer was a certified specialist in immigration law, as certified by the state bar association itself. While the certification was legally valid, the lawyer had not handled a single immigration case since obtaining the certification some time ago, and has not kept abreast of major changes in immigration law in the meantime. Given the extensiveness and complexity of recent changes in the law, the attorney would no longer be able to provide competent representation in immigration matters without extensive research and study. Last week, the lawyer happened to drive by two cars on the shoulder of the road that had been in a minor collision. The attorney pulled over, got out of her car, and approached one of the drivers who were waiting for a tow truck. Handing the driver her business card, the attorney offered to represent her in any litigation over the accident, and assured her that she would charge a fair rate, and the driver gladly took it and said she might have seen one of the lawyer's advertisements. The attorney was not aware at the time that the driver was an immigrant from Europe who was in the country on a temporary work visa. The driver visited the attorney a few days later for an initial consultation, but decided not to retain the lawyer for representation because the driver's insurer had already settled the matter. Was it proper for the lawyer to offer to represent the driver in this way?

D.No, because the lawyer approached the driver in person at the scene of the accident and offered to represent her.

QUESTION 4. A chance encounter between adversaries. The plaintiff and the defendant in a lawsuit run into each other in the supermarket and start discussing their case without their lawyers being present. Both have been shocked at the mounting litigation costs, and at how long the case has gone on. The plaintiff volunteers to withdraw his case if the defendant will withdraw his counterclaims and pay whatever filing fees are involved in such a voluntary dismissal. Later, when each party reports this to their respective lawyers, the plaintiff's lawyer is very upset. The plaintiff mentioned that the defendant said his own attorney (defense counsel) had helped give him the idea by asking at their first consultation, "Why haven't you and the plaintiff simply resolved this on your own, without resorting to litigation?" The plaintiff's lawyer reports the defendant's attorney for misconduct, claiming that opposing counsel merely used his client as an agent to communicate with the plaintiff without the latter's lawyer present. Is the defendant's attorney subject to discipline, based on these facts?

D.No, because the parties to a matter may communicate directly with each other without their lawyers being present or consenting to the conversation.

QUESTION 3. Attending the lawyer's party. A judge attended a social event—a birthday party—with around thirty other individuals at the home of a lawyer who had been a longtime acquaintance. The lawyer who hosted party in his home had a pending case before the judge that was awaiting a pretrial procedural ruling. Apart from exchanging brief greetings when the judge first arrived, the lawyer and the judge did not engage in any substantial conversation during the party, and neither the judge nor the lawyer discussed the pending case itself with anyone at the party. A month later, the judge ruled in favor of the party represented by the lawyer. Was it improper for the judge to attend the social event, given that it occurred at the home of a lawyer who had a case pending before the judge?

D.No, because there were no ex parte communications about the pending matter, and it was a social event attended by thirty people.

QUESTION 10. They will find out eventually. A government agency contacts an attorney, who works as in-house counsel for Corporation, and requests a report about some of Corporation's activities that come under the agency's regulatory jurisdiction. As the attorney begins to investigate the matter to prepare the report, he learns that the information requested by the agency will subject Corporation to significant regulatory enforcement sanctions, and if the information became public, would adversely affect Corporation's share price. At this point, the agency has not issued a subpoena and compliance with the request is voluntary, although the agency could compel the disclosure eventually. The managers and directors of Corporation instruct the attorney not to submit the report until the agency issues a subpoena, to buy some time to mitigate their regulatory violations. May the attorney prepare the report and submit it to the agency at this time?

D.No, because when a lawyer knows or reasonably should know that the evaluation is likely to affect the client's interests materially and adversely, the lawyer shall not provide the evaluation unless the client gives informed consent.

QUESTION 4. Test results. A client was with three friends in a car when a police officer stopped the vehicle. During the stop, the police officer found cocaine and marijuana in the vehicle. The prosecutor charged the client for possession of a controlled substance. The prosecutor did not charge anyone for possessing marijuana, though it was illegal to possess such a substance in the jurisdiction where the vehicle was stopped. The attorney knows the client uses marijuana. The client took a drug test at the attorney's recommendation. The drug test shows the client negative for cocaine but positive for marijuana. The attorney wants to use the drug test to show it was unlikely that the cocaine found in the car belonged to the client. Nevertheless, providing the drug test to the prosecutor would reveal that the client tested positive for marijuana and might lead to charges based on the marijuana found in the vehicle at the time of the stop. The attorney asks his client if he can show the prosecutor the drug test as evidence that the client did not use cocaine, but does not mention the risk of additional charges. The client tells the attorney he can share the results with the prosecutor. Did the attorney act properly?

D.No, the client must give informed consent and the attorney did not make the client aware of the risks and reasonable alternatives.

QUESTION 10. The prospective client. An attorney has been practicing law for two years, and has represented some law school graduates in their appeals before the bar when the Board of Law Examiners had denied the applications for licenses on character or fitness grounds. A former law school classmate who was a first-year student when the attorney was a third-year student visits the attorney in his office. The former classmate was on law review and graduated near the top of the class, but now he expresses concern about the character portion of the bar application. "I need you to represent me before the Board of Law Examiners," the former classmate said. The attorney asks the classmate to explain the problem. The classmate then explains a history of heroin addiction in college, which led to a criminal conviction and a period of incarceration; but a successful rehabilitation program enabled the student to beat this addiction and live drug-free throughout law school. The classmate does not want to disclose this on the bar application. The attorney declines to represent the former student, and later receives a call from the bar examiners inquiring about this former classmate's character and fitness. The attorney then recounts everything the classmate said about the past addiction and criminal conviction. Was the attorney's conduct proper in this situation?

D.No, the former classmate here was a prospective client, and the attorney owed a duty of confidentiality, even though no representation occurred.

QUESTION 27. Moonlighting and retiring. Big Firm has a strict rule against moonlighting or its lawyers working cases on the side, even on a pro bono basis. The rule, which it embodied in its employment contract with all the attorneys who work there, is partly to avoid conflicts of interest with these clients who are unknown to Big Firm's managing partners. On the other hand, there is a pure profit motive as well—Big Firm wants the fees from every hour of legal work its lawyers perform for any client whatsoever. Some of the associates question whether this rule might be improper. In addition, Big Firm's employment policies provide that after one of its lawyers retires, at age sixty-five, the firm will pay the former employee five thousand dollars per month for life, so long as the attorney does not re-enter law practice anywhere. This stipend would be in addition to whatever income the retiring lawyer has from a pension or other retirement investments. Which of the following is true regarding Big Firm's internal policies?

D.The policies are proper, for it is a universally recognized exception to the rule against restrictions on lawyer's right to practice law that legal employers can require lawyers to devote their entire practice to the firm's clients.

QUESTION 19. One time thing. In anticipation of a hearing before a federal agency in Washington, D.C., an attorney travels to a Washington suburb in Virginia from her own state to meet with her client (from her home state), interview witnesses, and review relevant documents. The attorney makes weekly trips there over the course of a year and spends most of her workweek there each time (four or five days), as the agency hearing pertains to a complex antitrust matter. The attorney solicits no new clients there. She works only on the matter for the client from her home state but is nonetheless unlicensed in Virginia. Is the attorney's conduct proper?

D.Yes, because a lawyer rendering services in a foreign jurisdiction on a temporary basis does not violate the rules merely by engaging in conduct in anticipation of a proceeding or hearing in a jurisdiction in which the lawyer is authorized to practice law.

QUESTION 23. Licensed in a neighboring state. An attorney has a license to practice in the state in which his firm operates. He hires as an associate, a law school friend who does not have a license in the state, who but holds a license to practice in a neighboring state with similar laws and precedents. The attorney gives the associate attorney only simple cases that require mostly scrivener's work (paperwork) for the clients, but he allows the associate to interview clients and to prepare and file client forms and paperwork. About once a week, the attorney checks with the associate and asks how his work is going, and the associate always says everything is fine, and occasionally asks questions about local laws or rules. Any clients whose matters seem to require actual litigation go to the attorney, and the associate handles only non-litigation forms and filings for clients. Is the attorney subject to discipline for this arrangement?

D.Yes, because the attorney is assisting another person in the unlicensed practice of law in his jurisdiction.

QUESTION 24. Eliminating the plaintiff's lawyer. An attorney is a notorious personal injury lawyer, widely feared by defendant corporations and insurers who must defend claims. The attorney reaches one exceptionally favorable settlement for his client, a structured settlement paying several hundred million dollars over the period of five years. The defendant has lost cases to the attorney on several occasions and wants to avoid dealing with him in the future. The defendant demands, as a condition of settlement, that the attorney will not represent any other clients in the future in tort actions related to this defendant or even to similar businesses in that jurisdiction. The attorney's contingent fee will be large enough for him to retire comfortably to a private tropical island and never need to work again, so he is amenable to this condition of the settlement. Is the attorney subject to discipline for this agreement?

D.Yes, the agreement violates the rules, but the attorney may not care about being subject to discipline if he plans to leave the practice of law.


Kaugnay na mga set ng pag-aaral

Biology Chapter 17 Gene Expression

View Set

Chapter 11 Homework: Cardiovascular System

View Set

Toxicity - Which vitamin and level of toxicity is the symptom associated with?

View Set

PHIL 100 CSU Alvarez, exam 5, Final

View Set